Last visit was: 18 Nov 2025, 22:22 It is currently 18 Nov 2025, 22:22
Close
GMAT Club Daily Prep
Thank you for using the timer - this advanced tool can estimate your performance and suggest more practice questions. We have subscribed you to Daily Prep Questions via email.

Customized
for You

we will pick new questions that match your level based on your Timer History

Track
Your Progress

every week, we’ll send you an estimated GMAT score based on your performance

Practice
Pays

we will pick new questions that match your level based on your Timer History
Not interested in getting valuable practice questions and articles delivered to your email? No problem, unsubscribe here.
Close
Request Expert Reply
Confirm Cancel
User avatar
Sajjad1994
User avatar
GRE Forum Moderator
Joined: 02 Nov 2016
Last visit: 18 Nov 2025
Posts: 17,289
Own Kudos:
49,293
 [27]
Given Kudos: 6,179
GPA: 3.62
Products:
Posts: 17,289
Kudos: 49,293
 [27]
2
Kudos
Add Kudos
25
Bookmarks
Bookmark this Post
User avatar
desertEagle
Joined: 14 Jun 2014
Last visit: 03 Aug 2025
Posts: 567
Own Kudos:
Given Kudos: 413
Posts: 567
Kudos: 344
Kudos
Add Kudos
Bookmarks
Bookmark this Post
User avatar
Sajjad1994
User avatar
GRE Forum Moderator
Joined: 02 Nov 2016
Last visit: 18 Nov 2025
Posts: 17,289
Own Kudos:
49,293
 [1]
Given Kudos: 6,179
GPA: 3.62
Products:
Posts: 17,289
Kudos: 49,293
 [1]
1
Kudos
Add Kudos
Bookmarks
Bookmark this Post
User avatar
Sajjad1994
User avatar
GRE Forum Moderator
Joined: 02 Nov 2016
Last visit: 18 Nov 2025
Posts: 17,289
Own Kudos:
49,293
 [1]
Given Kudos: 6,179
GPA: 3.62
Products:
Posts: 17,289
Kudos: 49,293
 [1]
1
Kudos
Add Kudos
Bookmarks
Bookmark this Post
Explanation

3. According to the passage, which one of the following is true?

Explanation

The question asks which answer is true according to statements in the passage. Since the answer could be supported from a statement located anywhere in the passage, look for a statement in the passage that would prove an answer choice true. Eliminate answers that contradict the Bottom Line or include strongly worded language or comparisons that are not supported by statements in the passage.

A. No. This answer contradicts the Bottom Line of the passage. The passage argues that the requirements of theoretical equipoise are effectively impossible for clinical trials to satisfy.

B. No. This answer goes beyond the passage. The passage does not discuss how often clinical researchers are forced to suspend trials in this manner.

C. Yes. This answer is supported by a statement in the passage. In lines 50–53, the passage states that even if one or more researchers has a decided clinical preference as to treatment, this situation would be no ethical bar to participation in a trial, which implies that a physician holding such a preference would not render the clinical trial unethical.

D. No. This answer contradicts the Bottom Line of the passage. The passage suggests that the standard of theoretical equipoise is too restrictive, and it proposes clinical equipoise as a less restrictive alternative; therefore, a clinical trial that meets the standard of clinical equipoise would not necessarily meet the standard of theoretical equipoise.

E. No. This answer contradicts the Bottom Line of the passage. The passage suggests that theoretical equipoise is the traditional standard applied to clinical trials, and it argues that a new standard of clinical equipoise should be adopted. However, the passage does not state that researchers already do try to conduct trials in accordance with the clinical equipoise standard.

Answer: C
User avatar
Sajjad1994
User avatar
GRE Forum Moderator
Joined: 02 Nov 2016
Last visit: 18 Nov 2025
Posts: 17,289
Own Kudos:
Given Kudos: 6,179
GPA: 3.62
Products:
Posts: 17,289
Kudos: 49,293
Kudos
Add Kudos
Bookmarks
Bookmark this Post
Explanation

5. Which one of the following most accurately expresses the main point of the passage?

Explanation

Use the Bottom Line to choose an answer. Watch out for answers that are too narrow, go too far, or contradict the Bottom Line of the passage.

A. No. This answer is too narrow. This is the main idea of the second paragraph only. It does not address the main ideas of the third and fourth paragraphs: that a new standard of clinical equipoise should be adopted instead.

B. No. This answer does not match the Bottom Line. While the passage does say in the second paragraph that the conception of theoretical equipoise is almost impossible to satisfy, the passage goes on in the third and fourth paragraphs to propose a new standard that should be adopted.

C. No. This answer is too narrow. This is the main idea of the fourth paragraph only. This answer does not address the main point, which is that the restrictive conception of theoretical equipoise should be replaced with the less restrictive conception of clinical equipoise.

D. Yes. This answer matches the Bottom Line. The second paragraph argues that theoretical equipoise is too restrictive, and the third and fourth paragraphs argue that clinical equipoise is less restrictive and therefore should be adopted.

E. No. This answer is too narrow. The passage argues that a clinical trial that does not meet the standard of theoretical equipoise but does meet the standard of clinical equipoise should not be considered unethical.

Answer: D
User avatar
Equinox1
Joined: 09 Dec 2021
Last visit: 01 Mar 2024
Posts: 4
Posts: 4
Kudos: 0
Kudos
Add Kudos
Bookmarks
Bookmark this Post
Pls post solutions to Qn 4 and 7
User avatar
Sajjad1994
User avatar
GRE Forum Moderator
Joined: 02 Nov 2016
Last visit: 18 Nov 2025
Posts: 17,289
Own Kudos:
49,293
 [1]
Given Kudos: 6,179
GPA: 3.62
Products:
Posts: 17,289
Kudos: 49,293
 [1]
1
Kudos
Add Kudos
Bookmarks
Bookmark this Post
Equinox1
Pls post solutions to Qn 4 and 7

Explanation

4. Suppose two medical treatments are being compared in a clinical trial for their effectiveness in treating a condition. Based on the passage, which one of the following scenarios would be significantly more likely to jeopardize theoretical equipoise than clinical equipoise?

Difficulty Level: 700

Explanation

The question asks which answer would be significantly more likely to jeopardize theoretical equipoise than clinical equipoise. Look for an answer that describes a violation of the standard of theoretical equipoise but that would comply with the standard of clinical equipoise. According to the second paragraph, theoretical equipoise requires that the researcher consider the evidence for the treatment regimens being compared to be exactly balanced. According to the third paragraph, clinical equipoise provides that a researcher may prefer one treatment over another based on evidence, so long as clinical experts disagree as to which treatment is superior and the researcher recognizes this lack of consensus.

A. No. This answer would not achieve the goal stated by the question. If, during a clinical trial, most clinical specialists came to favor one treatment over another, there would be no lack of consensus among medical experts, and therefore this scenario would violate the standard of clinical equipoise.

B. No. This answer would not achieve the goal stated by the question. If preliminary results indicate that the two treatments are equally effective, then researchers participating in the study would have no reason to prefer one treatment over another, and this scenario would not jeopardize theoretical equipoise.

C. No. This answer would not achieve the goal stated by the question. If physicians participating in the study prefer one treatment to another, then this scenario would jeopardize theoretical equipoise, but if there is no lack of consensus among clinical experts, then this scenario would also jeopardize clinical equipoise.

D. Yes. This answer achieves the goal stated by the question. If physicians participating in the study prefer one treatment to another, then this scenario would jeopardize theoretical equipoise, but if there is a lack of consensus among clinical experts and the physicians in question recognize this, then this scenario would not jeopardize clinical equipoise.

E. No. This answer would not achieve the goal stated by the question. If physicians participating in the study believe both treatments are equally effective, then this scenario would not jeopardize theoretical equipoise.

Answer: D
User avatar
Sajjad1994
User avatar
GRE Forum Moderator
Joined: 02 Nov 2016
Last visit: 18 Nov 2025
Posts: 17,289
Own Kudos:
Given Kudos: 6,179
GPA: 3.62
Products:
Posts: 17,289
Kudos: 49,293
Kudos
Add Kudos
Bookmarks
Bookmark this Post
Equinox1
Pls post solutions to Qn 4 and 7

Explanation

7. According to the passage, which one of the following is true?

Difficulty Level: 700

Explanation

The question asks which answer is true according to statements in the passage. Since the answer could be supported from a statement located anywhere in the passage, look for a statement in the passage that would prove an answer choice true. Eliminate answers that contradict the Bottom Line or include strongly worded language or comparisons that are not supported by statements in the passage.

A. No. This answer contradicts the passage. The last sentence of the second paragraph of the passage states that few trials could comply with the standard of theoretical equipoise.

B. No. This answer makes an unsupported comparison. The passage does not suggest that clinical trials would be conducted more often if a more reasonable standard were in place; rather, the passage suggests that more clinical trials would be able to satisfy that new standard than the current standard.

C. Yes. This answer is supported by a statement in the passage. The last sentence of the second paragraph of the passage states that few trials could comply with the standard of theoretical equipoise.

D. No. This answer goes too far. While the last sentence of the first paragraph does state that most physicians and ethicists have traditionally agreed that traditional equipoise is appropriate for physicians in clinical trials, the passage does go so far as to state that most of them believe the currently accepted ethical requirements are adequate.

E. No. This answer goes beyond the passage. While the third paragraph does discuss conflicts of opinion in the expert clinical community, the passage does not suggest that most comparative trials are undertaken to help resolve such conflicts of opinion.

Answer: C
User avatar
ARSarkar
Joined: 26 Nov 2022
Last visit: 08 Aug 2023
Posts: 31
Own Kudos:
29
 [1]
Given Kudos: 42
Location: India
GMAT 1: 700 Q49 V36
GMAT 1: 700 Q49 V36
Posts: 31
Kudos: 29
 [1]
Kudos
Add Kudos
1
Bookmarks
Bookmark this Post
Sajjad1994
Equinox1
Pls post solutions to Qn 4 and 7

Explanation

4. Suppose two medical treatments are being compared in a clinical trial for their effectiveness in treating a condition. Based on the passage, which one of the following scenarios would be significantly more likely to jeopardize theoretical equipoise than clinical equipoise?

Difficulty Level: 700

Explanation

The question asks which answer would be significantly more likely to jeopardize theoretical equipoise than clinical equipoise. Look for an answer that describes a violation of the standard of theoretical equipoise but that would comply with the standard of clinical equipoise. According to the second paragraph, theoretical equipoise requires that the researcher consider the evidence for the treatment regimens being compared to be exactly balanced. According to the third paragraph, clinical equipoise provides that a researcher may prefer one treatment over another based on evidence, so long as clinical experts disagree as to which treatment is superior and the researcher recognizes this lack of consensus.

A. No. This answer would not achieve the goal stated by the question. If, during a clinical trial, most clinical specialists came to favor one treatment over another, there would be no lack of consensus among medical experts, and therefore this scenario would violate the standard of clinical equipoise.

B. No. This answer would not achieve the goal stated by the question. If preliminary results indicate that the two treatments are equally effective, then researchers participating in the study would have no reason to prefer one treatment over another, and this scenario would not jeopardize theoretical equipoise.

C. No. This answer would not achieve the goal stated by the question. If physicians participating in the study prefer one treatment to another, then this scenario would jeopardize theoretical equipoise, but if there is no lack of consensus among clinical experts, then this scenario would also jeopardize clinical equipoise.

D. Yes. This answer achieves the goal stated by the question. If physicians participating in the study prefer one treatment to another, then this scenario would jeopardize theoretical equipoise, but if there is a lack of consensus among clinical experts and the physicians in question recognize this, then this scenario would not jeopardize clinical equipoise.

E. No. This answer would not achieve the goal stated by the question. If physicians participating in the study believe both treatments are equally effective, then this scenario would not jeopardize theoretical equipoise.

Answer: D


The passage says :
Quote:
Even if researchers judged the evidence to
be balanced at the start of a comparative clinical trial,
such a balance would be extremely fragile, liable to
be “tipped” by small accretions of evidence as the
study progresses. Consequently, if the standard of
(30) theoretical equipoise is adhered to, few comparative
clinical trials could commence and even fewer could
proceed to completion.
.

That means as the study progresses and some evidences come that may tilt the balance , theoretical equipoise cannot be achieved. So option B tells the same thing. So why it is incorrect. Please explain.
User avatar
Sajjad1994
User avatar
GRE Forum Moderator
Joined: 02 Nov 2016
Last visit: 18 Nov 2025
Posts: 17,289
Own Kudos:
49,293
 [1]
Given Kudos: 6,179
GPA: 3.62
Products:
Posts: 17,289
Kudos: 49,293
 [1]
1
Kudos
Add Kudos
Bookmarks
Bookmark this Post
ARSarkar
The passage says :
Quote:
Even if researchers judged the evidence to
be balanced at the start of a comparative clinical trial,
such a balance would be extremely fragile, liable to
be “tipped” by small accretions of evidence as the
study progresses. Consequently, if the standard of
(30) theoretical equipoise is adhered to, few comparative
clinical trials could commence and even fewer could
proceed to completion.
.

That means as the study progresses and some evidences come that may tilt the balance , theoretical equipoise cannot be achieved. So option B tells the same thing. So why it is incorrect. Please explain.

The difference between answer choices (B) and (D) is that in (B), preliminary results suggest that the treatments are equally effective but are not reported, while in (D), initial results convince some participating physicians that one treatment is more effective than the other. In (B), the lack of reporting may prevent the development of clinical equipoise, but in (D), the recognition of the lack of consensus among experts allows for the maintenance of clinical equipoise despite some researchers having a preference for one treatment over the other.
User avatar
a.k01
Joined: 26 Apr 2023
Last visit: 06 Oct 2023
Posts: 1
Given Kudos: 140
Posts: 1
Kudos: 0
Kudos
Add Kudos
Bookmarks
Bookmark this Post
Please, post solutions to Question 8
User avatar
Sajjad1994
User avatar
GRE Forum Moderator
Joined: 02 Nov 2016
Last visit: 18 Nov 2025
Posts: 17,289
Own Kudos:
49,293
 [1]
Given Kudos: 6,179
GPA: 3.62
Products:
Posts: 17,289
Kudos: 49,293
 [1]
1
Kudos
Add Kudos
Bookmarks
Bookmark this Post
a.k01
Please, post solutions to Question 8

Explanation

8. The author’s argument in the third and fourth paragraphs would be most weakened if which one of the following were true?

Difficulty Level: 700

Explanation

The question asks which answer, if true, would most weaken the author’s argument in the third and fourth paragraphs. Treat this question the same as a Weaken question in the Arguments section. The author’s conclusion in the third and fourth paragraphs is that a new standard called “clinical equipoise” should be developed. The author supports this conclusion with the premise that a physician participating in a clinical study who develops a preference for one treatment over another should be allowed to continue to participate so long as a lack of consensus exists among clinical experts, and the physician acknowledges this lack of consensus. Look for an answer that suggests a problem with this plan to develop a new standard of clinical equipoise.

A. Yes. This answer weakens the argument in the third and fourth paragraphs. If most comparative clinical trials are undertaken to prove that a treatment considered best by a consensus of relevant experts is superior, then the standard of clinical equipoise would be jeopardized, because the standard of clinical equipoise requires that a lack of consensus exist among clinical experts as to which treatment is superior.

B. No. This answer strengthens the argument. If physicians rarely ask to leave trials when they believe early data favors one treatment over another, then the study does not satisfy the requirement of theoretical equipoise. This supports the plan to develop a new standard of clinical equipoise that would allow those physicians to remain in the study so long as they acknowledge a lack of consensus among medical experts.

C. No. This answer is irrelevant. The number of clinical trials being conducted annually does not affect the decision whether to develop a new standard of clinical equipoise.

D. No. This answer is irrelevant. The opinion of medical ethicists compared with the opinion of clinical researchers is not relevant to whether a new standard clinical equipoise should be developed.

E. No. This answer is not strong enough to weaken the argument. Even if it is rare that researchers begin a trial with no preference, then later develop a strong preference, the standard of clinical equipoise would still be needed to deal with such situations; furthermore, the standard of clinical equipoise would also still be needed to deal with other situations, such as allowing researchers to participate in a trial even if they begin the trial with a preference for one treatment over another.

Answer: A
User avatar
Raman109
Joined: 17 Aug 2009
Last visit: 28 Jul 2025
Posts: 805
Own Kudos:
Given Kudos: 33
Posts: 805
Kudos: 170
Kudos
Add Kudos
Bookmarks
Bookmark this Post
­4. Suppose two medical treatments are being compared in a clinical trial for their effectiveness in treating a condition. Based on the passage, which one of the following scenarios would be significantly more likely to jeopardize theoretical equipoise than clinical equipoise?

Option Elimination - 

(A) The initial results of the trial so strikingly favored one treatment that they were published and widely disseminated before the study was even half over (Ok breaks the theoretical equipoise - TE); as a result, most physicians who specialize in treating the condition came to favor the more effective treatment before the trial had ended. (If they come to favor and there is consensus, that breaks the clinical equipoise - CE). We need an option that breaks TE and saves CE, but this option breaks both. Wrong. 

(B) Preliminary results in the trial suggest that the two treatments are equally effective in treating the condition (Doesn't break TE), but these results are not reported while the trial is underway, and thus, few in the expert clinical community are aware of them (few are practically none. This doesn't have much impact on CE, as there should be no consensus on whether CE should pass the "ethical" criteria. Being unaware may mean they have preferences or no preferences. We don't know. Wrong. 

(C) Several of the physicians participating in the trial think that one treatment is more effective at treating the condition than the other (breaks TE); in this, they agree with the consensus view within the expert clinical community. (The only condition for CE is no consensus, so if they have consensus, it breaks CE as well) - Wrong. 

(D) Initial results from the trial convince several of the participating physicians that one treatment more effectively treats the condition than the other does (breaks TE as there is no 50/50 chance); this does not affect their recognition of the lack of consensus among experts in treating the disease. (This means they still lack consensus, which is the only condition for CE to prevail. Doesn't break CE) - ok.

(E) There is consensus among physicians participating in the trial that both treatments are equally effective at treating the condition (Doesn't break TE); however, there is no consensus within the expert medical community as to the relative effectiveness of the treatments. (Doesn't break CE) - Wrong as it doesn't break TE. 
User avatar
Raman109
Joined: 17 Aug 2009
Last visit: 28 Jul 2025
Posts: 805
Own Kudos:
Given Kudos: 33
Posts: 805
Kudos: 170
Kudos
Add Kudos
Bookmarks
Bookmark this Post
8. The author’s argument in the third and fourth paragraphs would be most weakened if which one of the following were true?

(A) In most comparative clinical trials, the main purpose is to prove definitively that a treatment considered best by a consensus of relevant experts is, in fact, superior to the alternative being tested. - No. If there is consensus in the beginning and we still conduct a trial, it's unethical. The basic or the only condition for CE is that there is no consensus initially. This breaks the argument for CE. 

(B) Physicians participating in comparative clinical trials rarely ask to leave the trials because early data favors one of the treatments being tested over another. - This weakens the TE and not CE. Wrong. 

(C) The number of comparative clinical trials that are conducted annually is increasing rapidly, but the level of ethical oversight of these trials is decreasing. - Out of scope. There is no compromise on ethics in either. CE just removes the 50/50 constraint from TE. 

(D) Medical ethicists are more inclined than are clinical researchers to favor an ethical requirement based on theoretical equipoise over one based on clinical equipoise. - There is no compromise on ethics by any group. Wrong. 

(E) In clinical trials comparing two treatments, it rarely occurs that researchers who begin the trial with no preference for either of the treatments later develop a strong preference on the basis of data obtained early in the study.­ - while it strengthens TE, it has no impact on CE. Out of scope. 
User avatar
nikitathegreat
Joined: 16 Dec 2021
Last visit: 18 Nov 2025
Posts: 201
Own Kudos:
Given Kudos: 110
Location: India
GMAT 1: 630 Q45 V31
Products:
GMAT 1: 630 Q45 V31
Posts: 201
Kudos: 22
Kudos
Add Kudos
Bookmarks
Bookmark this Post
GMATNinja - Can you explain why Q2 option choice A is incorrect? Doesnt it provides a view that contrasts with the later paras?

2. The primary purpose of the second paragraph of the passage is to

(A) provide a view that contrasts with arguments in favor of clinical equipoise
(B) explore the factors underlying physicians’ preferences regarding competing treatments
(C) undermine the moral principle that underlies the theory of theoretical equipoise
(D) state the main difficulty with adhering to the standards of theoretical equipoise
(E) illustrate the conflicts inherent in the general notion of equipoise

@VeritasKarishma
User avatar
GMATNinja
User avatar
GMAT Club Verbal Expert
Joined: 13 Aug 2009
Last visit: 18 Nov 2025
Posts: 7,445
Own Kudos:
Given Kudos: 2,060
Status: GMAT/GRE/LSAT tutors
Location: United States (CO)
GMAT 1: 780 Q51 V46
GMAT 2: 800 Q51 V51
GRE 1: Q170 V170
GRE 2: Q170 V170
Products:
Expert
Expert reply
GMAT 2: 800 Q51 V51
GRE 1: Q170 V170
GRE 2: Q170 V170
Posts: 7,445
Kudos: 69,779
Kudos
Add Kudos
Bookmarks
Bookmark this Post

Question 2


nikitathegreat
[url=https://gmatclub.com:443/forum/memberlist.php?mode=viewprofile&un=GMATNinja%5D%5Bb%5DGMATNinja%5B/b%5D%5B/url%5D  - Can you explain why Q2 option choice A is incorrect? Doesnt it provides a view that contrasts with the later paras?

The primary purpose of the second paragraph of the passage is to

(A) provide a view that contrasts with arguments in favor of clinical equipoise
(B) explore the factors underlying physicians’ preferences regarding competing treatments
(C) undermine the moral principle that underlies the theory of theoretical equipoise
(D) state the main difficulty with adhering to the standards of theoretical equipoise
(E) illustrate the conflicts inherent in the general notion of equipoise
 
The point of the paragraph is that "clinical equipoise" is unworkable, which is why we need an alternative equipoise, one that's explored in the following paragraph. So it's actually expressing a view that's fully consistent with later paragraphs.

If the second paragraph had been about how awesome clinical equipoise was, well, THAT would contrast with ideas expressed later. But that's not what we get, so (A) is incorrect.

I hope that helps!
User avatar
RenB
Joined: 13 Jul 2022
Last visit: 18 Nov 2025
Posts: 391
Own Kudos:
Given Kudos: 303
Location: India
Concentration: Finance, Nonprofit
GMAT Focus 1: 715 Q90 V84 DI82
GPA: 3.74
WE:Corporate Finance (Consulting)
Kudos
Add Kudos
Bookmarks
Bookmark this Post
­Hi HarshR9, can you please have a look at qs 3? I am not able to figure out why C is correct. I chose A, though I had a hunch that there is a possibility that though the trial may be ethical as per the considerations in this qs, the trial may be unethical due to other reasons. But couldntfigure out why C is correct thus chose A.
User avatar
HarshavardhanR
Joined: 16 Mar 2023
Last visit: 18 Nov 2025
Posts: 425
Own Kudos:
460
 [1]
Given Kudos: 59
Status:Independent GMAT Tutor
Affiliations: Ex - Director, Subject Matter Expertise at e-GMAT
Expert
Expert reply
Active GMAT Club Expert! Tag them with @ followed by their username for a faster response.
Posts: 425
Kudos: 460
 [1]
1
Kudos
Add Kudos
Bookmarks
Bookmark this Post
 
RenB
­Hi HarshR9, can you please have a look at qs 3? I am not able to figure out why C is correct. I chose A, though I had a hunch that there is a possibility that though the trial may be ethical as per the considerations in this qs, the trial may be unethical due to other reasons. But couldntfigure out why C is correct thus chose A.
­Sure.

(C) A clinical trial comparing treatments is not rendered unethical merely because one of the participating physicians has come to favor one of the treatments over the other.

The logic you have presented contains the answer to your question.

I hope that it is clear (from the passage) that a clinical trial comparing treatments could still be ethical even if some of the participating physicians prefer one of the treatments. That is the crux of what "clinical equipoise" achieves - it is ok for doctors to have their own preferences and prescribe a treatment to a patient based on their preferences (accepted/establised vs. under test), as long as they acknowledge that the other treatment may also be preferred by a sizable chunk of doctors.

So, a clinical trial comparing treatments could still be ethical even if some of the participating physicians prefer one of the treatments.
Of course, a trial may still end up being unethical due to other reasons. No denying that.

BUT - choice C merely says -> a trial does not become unethical because a doctor favours one treatment over another. A trial could become unethical due to "n" other reasons. But the passage clearly suggests that a trial will not be considered unethical for the reason that a doctor favors one treatment over the other.

Hope the nuance here is clear! 

Choice A is not really true. It actually goes against what the passage really says in my view. Theoretical equipoise creates an ethical conundrum which actually leads to a situation where few if any trials would ever get done.

"Consequently, if the standard of
(30) theoretical equipoise is adhered to, few comparative
clinical trials could commence and even fewer could
proceed to completion."

In other words, if theoretical equipose was adhered to, thanks to the ethics-related conundrums, few trials would even begin, very few would actually get done. Clearly, there are ethical problems being posed by theoretical equipoise here.

Cheers,
Harsha
Enthu about all things GMAT | Exploring the GMAT space | My website: gmatanchor.com
 
User avatar
RenB
Joined: 13 Jul 2022
Last visit: 18 Nov 2025
Posts: 391
Own Kudos:
Given Kudos: 303
Location: India
Concentration: Finance, Nonprofit
GMAT Focus 1: 715 Q90 V84 DI82
GPA: 3.74
WE:Corporate Finance (Consulting)
Kudos
Add Kudos
Bookmarks
Bookmark this Post
HarshR9

RenB
­Hi HarshR9, can you please have a look at qs 3? I am not able to figure out why C is correct. I chose A, though I had a hunch that there is a possibility that though the trial may be ethical as per the considerations in this qs, the trial may be unethical due to other reasons. But couldntfigure out why C is correct thus chose A.
­Sure.

(C) A clinical trial comparing treatments is not rendered unethical merely because one of the participating physicians has come to favor one of the treatments over the other.

The logic you have presented contains the answer to your question.

I hope that it is clear (from the passage) that a clinical trial comparing treatments could still be ethical even if some of the participating physicians prefer one of the treatments. That is the crux of what "clinical equipoise" achieves - it is ok for doctors to have their own preferences and prescribe a treatment to a patient based on their preferences (accepted/establised vs. under test), as long as they acknowledge that the other treatment may also be preferred by a sizable chunk of doctors.

So, a clinical trial comparing treatments could still be ethical even if some of the participating physicians prefer one of the treatments.
Of course, a trial may still end up being unethical due to other reasons. No denying that.

BUT - choice C merely says -> a trial does not become unethical because a doctor favours one treatment over another. A trial could become unethical due to "n" other reasons. But the passage clearly suggests that a trial will not be considered unethical for the reason that a doctor favors one treatment over the other.

Hope the nuance here is clear! 

Choice A is not really true. It actually goes against what the passage really says in my view. Theoretical equipoise creates an ethical conundrum which actually leads to a situation where few if any trials would ever get done.

"Consequently, if the standard of
(30) theoretical equipoise is adhered to, few comparative
clinical trials could commence and even fewer could
proceed to completion."

In other words, if theoretical equipose was adhered to, thanks to the ethics-related conundrums, few trials would even begin, very few would actually get done. Clearly, there are ethical problems being posed by theoretical equipoise here.

Cheers,
Harsha
Enthu about all things GMAT | Exploring the GMAT space | My website: gmatanchor.com

 
­Hi HarshR9,
Thank yo for the explanation.
For C, I thought, how do I know whether the option/ clinical trials considered are following the clinical equipose method? I thought that if the qs considers the theoretical equipose approach, then the option will be incorrect, if it follows clinical equipose, then yes, the option is correct.
Can you let me know where I am faltering here?
User avatar
HarshavardhanR
Joined: 16 Mar 2023
Last visit: 18 Nov 2025
Posts: 425
Own Kudos:
Given Kudos: 59
Status:Independent GMAT Tutor
Affiliations: Ex - Director, Subject Matter Expertise at e-GMAT
Expert
Expert reply
Active GMAT Club Expert! Tag them with @ followed by their username for a faster response.
Posts: 425
Kudos: 460
Kudos
Add Kudos
Bookmarks
Bookmark this Post
RenB

HarshR9

RenB
­Hi HarshR9, can you please have a look at qs 3? I am not able to figure out why C is correct. I chose A, though I had a hunch that there is a possibility that though the trial may be ethical as per the considerations in this qs, the trial may be unethical due to other reasons. But couldntfigure out why C is correct thus chose A.
­Sure.

(C) A clinical trial comparing treatments is not rendered unethical merely because one of the participating physicians has come to favor one of the treatments over the other.

The logic you have presented contains the answer to your question.

I hope that it is clear (from the passage) that a clinical trial comparing treatments could still be ethical even if some of the participating physicians prefer one of the treatments. That is the crux of what "clinical equipoise" achieves - it is ok for doctors to have their own preferences and prescribe a treatment to a patient based on their preferences (accepted/establised vs. under test), as long as they acknowledge that the other treatment may also be preferred by a sizable chunk of doctors.

So, a clinical trial comparing treatments could still be ethical even if some of the participating physicians prefer one of the treatments.
Of course, a trial may still end up being unethical due to other reasons. No denying that.

BUT - choice C merely says -> a trial does not become unethical because a doctor favours one treatment over another. A trial could become unethical due to "n" other reasons. But the passage clearly suggests that a trial will not be considered unethical for the reason that a doctor favors one treatment over the other.

Hope the nuance here is clear! 

Choice A is not really true. It actually goes against what the passage really says in my view. Theoretical equipoise creates an ethical conundrum which actually leads to a situation where few if any trials would ever get done.

"Consequently, if the standard of
(30) theoretical equipoise is adhered to, few comparative
clinical trials could commence and even fewer could
proceed to completion."

In other words, if theoretical equipose was adhered to, thanks to the ethics-related conundrums, few trials would even begin, very few would actually get done. Clearly, there are ethical problems being posed by theoretical equipoise here.

Cheers,
Harsha
Enthu about all things GMAT | Exploring the GMAT space | My website: gmatanchor.com


 
­Hi HarshR9,
Thank yo for the explanation.
For C, I thought, how do I know whether the option/ clinical trials considered are following the clinical equipose method? I thought that if the qs considers the theoretical equipose approach, then the option will be incorrect, if it follows clinical equipose, then yes, the option is correct.
Can you let me know where I am faltering here?
­Hi RenB,

Question Stem: As per the passage (i.e., as per the author of the passage), what is true?

Important: Statement C talks about something.

Given the author's statements on equipoise (theoretical, clinical, everything!) and everything else in the passage in fact, is that something true as per the author? - this is what we should be determining here. We are not being asked if the statement is true for theoretical or if the statement is true for clinical or anything of this sort. We are being asked if the statement is true as per the passage overall, given the author's statements. 

So, let's focus on what the author tells us. 

What the author (passage) says ->
Theoretical Equipoise -> is the concept of equipoise that is typically considered by people when thinking about equipoise. In other words, theoretical is what folks have in mind when they think of equipoise. As per the author, it is not practical. If it was properly applied, very few trials would actually happen. 
Clinical Equipoise -> is the concept of equipoise the author is trying to suggest as the one that is practical, and that should be developed in physicians, ethicists, etc. 

-> As per the author, clinical equipoise is the correct equipoise.
-> And so, as per the author, it is pefectly fine (i.e., still ethical) if one or more of the physicians have a preference for one of the treatments over the other, as long as they acknowledge that the other treatment is also preferred by a sizable number of physicians.
-> And so, as per the author, a clinical trial comparing treatments is not rendered unethical merely because one of the participating physicians has come to favor one of the treatments over the other.

The author's view is clinical equipoise. So, what clinical equipoise suggests is what is true as per the passage here. 

Hope this helps. 
___
Harsha
Enthu about all things GMAT | Exploring the GMAT space | My website: gmatanchor.com
 1   2   
Moderators:
GMAT Club Verbal Expert
7445 posts
GMAT Club Verbal Expert
234 posts
GRE Forum Moderator
17289 posts
188 posts